User Avatar
annette15lee452
Joined
Apr 2025
Subscription
Free

Obviously parallel flaw questions and some other question types like SA and MBT usually require logic mapping, but how do you decide during PTs and the actual test when to need to use conditional logic?

And what are the question types that 100% require mapping, often require mapping, and usually do not require mapping?

Thanks!

I am drilling with sections of older prep tests (currently focusing on LR) by printing the test and working on the hard copy. I have since completed the section but I would like to review my answers and watch any explanation videos needed. How do I do so if I did not input my answers/complete the whole test on 7Sage's platform? Hope this question makes sense. Thanks!

PrepTests ·
PT143.S3.Q9
User Avatar
annette15lee452
Wednesday, Mar 20 2024

Another reason why I think (E) is correct is even though they said "either," the later premise counted the 2 factors (troop movements & transfer of weapons) as one when they said that there's no indication of either. So the "either" didn't pose a problem.

PrepTests ·
PT118.S4.Q15
User Avatar
annette15lee452
Tuesday, Mar 19 2024

I was going to pick (A) but crossed it out because the stimulus says that the cost of tuition is based on number of courses a student takes but didn't say cost of tuition is based ONLY on a number of courses. So I thought maybe faculty salary increase could lead to an increase in tuition.

Like if my mood is based on my sleep quality, but maybe a shitty PT can also ruin my mood. You know what I mean?

I feel like a lot of LR questions do zone in on these super subtle words such as "only" "only if" blah blah and you can get a question wrong for assuming # of courses taken is the only factor that affects tuition

#help

PrepTests ·
PT135.S4.Q2
User Avatar
annette15lee452
Monday, Mar 18 2024

Stimulus:

- Conclusion: Syndicated political columnists' efforts to persuade voters to vote a certain way [using their newspaper columns] rarely succeed.

- Premise: Since nearly all voters will have already made a decision to vote for a certain candidate by the time the news paper column is published.

Necessary assumption?

(A) is so irrelevant. The argument focuses on voters who have already made decisions and the fact that these columns can't fail to convince voters. Why should we care about how columnists influence the votes of voters who have no clue who to vote for?

(B) is also super irrelevant. Whether or not the column (let's say it is trying to convince voters to vote for getting rid of daylight savings) can end up convincing voters to vote for keeping daylight savings DOES NOT wreck the argument!

(C) is similar to (B) in that this is irrelevant because who cares if a democrat reads CNN and a republican reads FOX? The argument of not being able to convince voters who have made decisions still stands.

(D) is also irrelevant. Who cares who reads and who doesn't read these columns at hand? This does not wreck our argument at all.

(E) is perfect. If people OFTEN can be persuaded once they have made their decisions then that means the columns are ABLE to influence the voters who have made their decisions. YAY!

PrepTests ·
PT135.S1.Q21
User Avatar
annette15lee452
Monday, Mar 18 2024

The stimulus is saying the govt incentives are helping to hasten global warming because the incentives focus on encouraging people to plant trees when research shows that native grasses absorb and store CO2 more effectively than trees.

Sure but why would this mean the incentives are hastening global warming? Vegetation such as trees are still helping absorb the CO2 that contributes to global warming, so how can one say the trees are making global warming worse than it needs to be? Does it have something to do with the effectiveness of native grasses? Does it have something to do with using trees in lieu of more effective methods?

(A) It doesn't matter whether trees not only absorb CO2 but also emit it. For all we know, native grasses may emit CO2 whilst absorbing it too. This isn't necessary.

(B) I don't care if most farmers don't plant trees unless there are incentives.

(C) Does deforested land that can't ever sustain native grasses lead to hastening global warming? This does not have to be necessary!

(D) This AC addresses the use of a less effective plant (trees) over a more effective plant (native grasses). If we are choosing to use trees (for example - absorb 60%, emit 40%) instead of native grasses (absorb 80%, emit 20%), then yes the incentive of planting trees instead of planting native grasses IS hastening global warming.

(E) I don't care.

PrepTests ·
PT135.S1.Q19
User Avatar
annette15lee452
Monday, Mar 18 2024

Stimulus:

- Premise(s): mild winter led to bird foraging naturally, aka no need to visit feeders; mild winter also led to birds not needing to migrate south, which led to the natural occurrence of population decrease in bird populations that accompany migration.

- Conclusion: Mild winter is responsible for the increased amount of birds

A) doesn't strengthen because it doesn't strengthen the relationship between the premises and conclusion. Also, "increases...SOMETIMES occur following unusual weather..."? If it sometimes occurs, then how can we conclude the mild winter is actually responsible? Maybe something else like the extinction of an animal that preys on birds led to the increase in population.

B) mentions "mating behaviors." But who cares? We don't even know what their mating behaviors are during a normal winter, and we definitely don't know how it has changed due to the mild winter.

C) is perfect. Premise 2 (no migration and no natural decrease due to migration) is pretty much set, so (C) actually solely supports Premise 1 (foraging naturally aka no visits to feeders). Why is foraging naturally preventing decreases in bird population? Maybe it's because predators wait for birds to go to feeders to hunt. Now with (C), both premises offer much stronger info to support the conclusion - YAY!

D) kind of weakens the argument, if anything.

E) umm... literally who cares?

PrepTests ·
PT137.S4.Q17
User Avatar
annette15lee452
Saturday, Mar 16 2024

A) is wrong because consumers can still save ton of money my using coupons at a given store. It isn't saying "a consumer will likely not save money using coupons in store providing coupons versus shopping at a store that doesn't provide coupons and charges less money for the same products."

B) is correct because this was basically stated in the stimulus.

C) is incorrect because we don't know about the costs of the other stores. We only know about the costs and profits of this store that provides coupons

D) is incorrect because the stimulus doesn't mention anything about other stores' costs and whether or not they pass some of it onto the customers. We only know about the coupon stores' costs and what they do to compensate for it.

E) is incorrect because the stimulus says "generally" the couponed price at store with coupons is still more expensive than regular price at other stores. This AC is basically saying no matter what the couponed price at store with coupons is still going to be more expensive than regular price at other stores.

PrepTests ·
PT137.S4.Q20
User Avatar
annette15lee452
Saturday, Mar 16 2024

Yup, misread AC C... :|

PrepTests ·
PT133.S3.Q21
User Avatar
annette15lee452
Friday, Mar 15 2024

The argument is concluding that the data on the weed killer's effects is probably misleading. So we're looking for an answer to strengthen the relationship between the premise and the probability that the data is misleading.

A) no one cares about the molecules and how they are unaffected by each other

B) perfectly strengthens the argument by saying since the stimulus tells us that in there is no perfect balance between the concentration of the two molecule forms, the data collected from the study are unreliable because the soil used in the study has a perfect balance of concentration of the two molecule forms. Basically a lab-created study with a perfectly controlled environment isn't reliable enough since the real world isn't perfect.

C) this literally has nothing to do with the data being misleading

D) opposite of B) in my opinion. This would likely weaken the argument!

E) Huh?

PrepTests ·
PT133.S3.Q17
User Avatar
annette15lee452
Friday, Mar 15 2024

Stimulus:

T --> /SP --> /DSI

T --> O

T --> R&A

---------------------

T --> /E

So the sufficient assumption would be any of the following options in order for the conclusion to be validly drawn:

Option 1: /DSI --> /E

Option 2: O --> /E

Option 3: R&A --> /E

A) This says DSI --> GE. Wtf?

B) This says R&A --> E. This is totally wrong! We're looking for R&A --> /E.

C) This says SP --> R&A. Uhh... How tf one would connect these two. And even if one did connect them, it wouldn't help with the conclusion.

D) This says /DSI --> /E. Yay, this is exactly what we're looking for!

E) This says /T --> /O. Wtf?

PrepTests ·
PT101.S3.Q23
User Avatar
annette15lee452
Tuesday, Aug 08 2023

Might be a dumb question lol, but when do we know we are looking for an answer that competes with an alternative hypothesis?

I feel like finding the strengthening (or except) answers have been relatively straightforward in these problem sets, but I'm having trouble understanding why on certain questions JY would know to find the answer that challenges an alternative hypothesis.

Hope my question makes sense.. Thanks in advance for your help!

#help (Added by Admin)

PrepTests ·
PT101.S2.Q20
User Avatar
annette15lee452
Tuesday, Aug 01 2023

I'm dumb lol literally forgot this was an EXCEPT question...

Confirm action

Are you sure?